2014 dxdy logo

Научный форум dxdy

Математика, Физика, Computer Science, Machine Learning, LaTeX, Механика и Техника, Химия,
Биология и Медицина, Экономика и Финансовая Математика, Гуманитарные науки


Правила форума


В этом разделе нельзя создавать новые темы.

Если Вы хотите задать новый вопрос, то не дописывайте его в существующую тему, а создайте новую в корневом разделе "Помогите решить/разобраться (М)".

Если Вы зададите новый вопрос в существующей теме, то в случае нарушения оформления или других правил форума Ваше сообщение и все ответы на него могут быть удалены без предупреждения.

Не ищите на этом форуме халяву, правила запрещают участникам публиковать готовые решения стандартных учебных задач. Автор вопроса обязан привести свои попытки решения и указать конкретные затруднения.

Обязательно просмотрите тему Правила данного раздела, иначе Ваша тема может быть удалена или перемещена в Карантин, а Вы так и не узнаете, почему.



Начать новую тему Ответить на тему На страницу Пред.  1, 2, 3, 4, 5, 6  След.
 
 Re: Функция Римана: "обратные" свойства.
Сообщение16.11.2005, 14:05 
Заслуженный участник
Аватара пользователя


23/07/05
17973
Москва
Anonymous писал(а):
За объяснение конечно же БОЛЬШОЕ СПАСИБО, но нельзя ли чуточку по подробнее написать. Я нефига не понял причём сдесь $a_2=\alpha_2+\frac{\beta_2-\alpha_2}{3}$ и $b_2=a_2+\min\{\frac{\beta_2-\alpha_2}{3},\frac{1}{2}\}$! Я спрашивал про отрезки [a1,b1] ; [a2,d2] ; [/math]


Так я Вам и объясняю, откуда берутся отрезки $[a_1,b_1]$, $[a_2,b_2]$,... Просто привожу конкретный способ (совершенно необязательный). Интервал $(\alpha_n,\beta_n)$ получается из определения нигде не плотного множества, а нам нужно выбрать какой-нибудь отрезок $[a_n,b_n]$ внутри него. Я разделил интервал $(\alpha_n,\beta_n)$ на три равные части и взял среднюю часть, тогда $a_n=\alpha_n+\frac{\beta_n-\alpha_n}{3}$ и $b_n=a_n+\frac{\beta_n-\alpha_n}{3}$. Но нам нужно, чтобы длины отрезков стремились к нулю, потому что мы хотим применить к ним теорему о стягивающихся отрезках. В данном случае нетрудно доказать (я это показывал), что длины действительно стремятся к нулю, но это требует определённых рассуждений. Чтобы этих рассуждений избежать, можно заранее ограничить длину отрезка $[a_n,b_n]$ величиной $\frac{1}{n}$, которая стремится к нулю. Для этого нужно написать $b_n=a_n+\min\{\frac{\beta_n-\alpha_n}{3},\frac{1}{n}\}$.

 Профиль  
                  
 
 
Сообщение16.11.2005, 14:25 
Заслуженный участник
Аватара пользователя


23/07/05
17973
Москва
PUZATIK писал(а):
А можно данную задачку по другому докoзать, или это единственное докозательство?
Если можно то пожалуйста напишите как.


Какую "данную"? О перечислении рациональных чисел? Можно. Например, так.
Определим множество $Q_n$ как множество рациональных чисел вида $\frac{m}{n}$, где $m\in\mathbb Z$. Так как множество целых чисел $\mathbb Z$ счётно, получаем, что и каждое $Q_n$, $n\in\mathbb N$ счётно. По теореме об объединении счётного множества счётных множеств, множество рациональных чисел $\bigcup_{n\in\mathbb N}Q_n$ счётно и, следовательно, его можно занумеровать натуральными числами (по определению счётного множества).
Но при таком доказательстве мы никакого конкретного способа нумерации не получаем - в отличие от изложенного ранее.

 Профиль  
                  
 
 Re: Функция Римана: "обратные" свойства.
Сообщение16.11.2005, 17:34 
Someone писал(а):
Anonymous писал(а):
За объяснение конечно же БОЛЬШОЕ СПАСИБО, но нельзя ли чуточку по подробнее написать. Я нефига не понял причём сдесь $a_2=\alpha_2+\frac{\beta_2-\alpha_2}{3}$ и $b_2=a_2+\min\{\frac{\beta_2-\alpha_2}{3},\frac{1}{2}\}$! Я спрашивал про отрезки [a1,b1] ; [a2,d2] ; [/math]


Так я Вам и объясняю, откуда берутся отрезки $[a_1,b_1]$, $[a_2,b_2]$,... Просто привожу конкретный способ (совершенно необязательный). Интервал $(\alpha_n,\beta_n)$ получается из определения нигде не плотного множества, а нам нужно выбрать какой-нибудь отрезок $[a_n,b_n]$ внутри него. Я разделил интервал $(\alpha_n,\beta_n)$ на три равные части и взял среднюю часть, тогда $a_n=\alpha_n+\frac{\beta_n-\alpha_n}{3}$ и $b_n=a_n+\frac{\beta_n-\alpha_n}{3}$. Но нам нужно, чтобы длины отрезков стремились к нулю, потому что мы хотим применить к ним теорему о стягивающихся отрезках. В данном случае нетрудно доказать (я это показывал), что длины действительно стремятся к нулю, но это требует определённых рассуждений. Чтобы этих рассуждений избежать, можно заранее ограничить длину отрезка $[a_n,b_n]$ величиной $\frac{1}{n}$, которая стремится к нулю. Для этого нужно написать $b_n=a_n+\min\{\frac{\beta_n-\alpha_n}{3},\frac{1}{n}\}$.


Большое спасибо! :D :D :D теперь то я всё понял!!!!!!!!!!!!

  
                  
 
 
Сообщение16.11.2005, 17:37 
Заслуженный участник


28/10/05
1368
А я думала, что Гость все это время прикалывается. Оказалось все не так просто. А индюк думал..

 Профиль  
                  
 
 
Сообщение16.11.2005, 17:39 
Someone писал(а):
PUZATIK писал(а):
А можно данную задачку по другому докoзать, или это единственное докозательство?
Если можно то пожалуйста напишите как.


Какую "данную"? О перечислении рациональных чисел? Можно. Например, так.
Определим множество $Q_n$ как множество рациональных чисел вида $\frac{m}{n}$, где $m\in\mathbb Z$. Так как множество целых чисел $\mathbb Z$ счётно, получаем, что и каждое $Q_n$, $n\in\mathbb N$ счётно. По теореме об объединении счётного множества счётных множеств, множество рациональных чисел $\bigcup_{n\in\mathbb N}Q_n$ счётно и, следовательно, его можно занумеровать натуральными числами (по определению счётного множества).
Но при таком доказательстве мы никакого конкретного способа нумерации не получаем - в отличие от изложенного ранее.


Я понимаю, что я задал не корректно поставленный вопрос! Вообще-то я имел ввиду:
"можно ли доказать, что не существует такой функции которая была бы непрерывной во всех рациональных точках и .... подругому? Или это единственный способ?

  
                  
 
 
Сообщение16.11.2005, 17:44 
LynxGAV писал(а):
А я думала, что Гость все это время прикалывается. Оказалось все не так просто. А индюк думал..


А счего это я должен прикалываться? Просто мне были непонятны многие моменты!!!
Задача очень интересная, и мне хочется хорошо понять как она решается! :D :D :D

Спасибо всем кто мне помогает разобраться!!!!!!!!!!!!!!!!

  
                  
 
 
Сообщение16.11.2005, 18:11 
Заслуженный участник


28/10/05
1368
Извините, PUZATIK. Была не права. У Вас столько смайлов и восклицательных знаков, что я растерялась.
:D Давно у меня не было столько эмоций.

 Профиль  
                  
 
 
Сообщение16.11.2005, 20:07 
Заслуженный участник
Аватара пользователя


23/07/05
17973
Москва
PUZATIK писал(а):
Я понимаю, что я задал не корректно поставленный вопрос! Вообще-то я имел ввиду:
"можно ли доказать, что не существует такой функции которая была бы непрерывной во всех рациональных точках и .... подругому? Или это единственный способ?


Не слишком ли много Вы от меня хотите?
Вы поняли то решение, которое я предложил? На мой взгляд, оно весьма естественно. Во всяком случае, оно сразу же пришло мне в голову.
Что касается другого способа, то он наверняка существует. Вот и попробуйте его придумать.

 Профиль  
                  
 
 Re: Функция Римана (обратные свойства)
Сообщение17.11.2005, 20:15 
Someone писал(а):
Множество $U_n$, очевидно, открыто; кроме того, оно является всюду плотным, так как содержит множество точек непрерывности функции $f(x)$. Поэтому дополнительное множество $F_n=\nathbb{R}\setminus U_n$ замкнуто и нигде не плотно на числовой прямой $\mathbb{R}$.



Почему если множество $U_n$ открыто и всюду плотно, то его дополнение $F_n=\nathbb{R}\setminus U_n$ замкнуто -- это я понял! А вот почему оно является нигде не плотным я не понял! Подскажите пожалуйста!!!! :cry: :cry: :cry:

  
                  
 
 Re: Функция Римана (обратные свойства)
Сообщение17.11.2005, 20:41 
Заслуженный участник
Аватара пользователя


23/07/05
17973
Москва
POKEMON писал(а):
Someone писал(а):
Множество $U_n$, очевидно, открыто; кроме того, оно является всюду плотным, так как содержит множество точек непрерывности функции $f(x)$. Поэтому дополнительное множество $F_n=\nathbb{R}\setminus U_n$ замкнуто и нигде не плотно на числовой прямой $\mathbb{R}$.



Почему если множество $U_n$ открыто и всюду плотно, то его дополнение $F_n=\nathbb{R}\setminus U_n$ замкнуто -- это я понял! А вот почему оно является нигде не плотным я не понял! Подскажите пожалуйста!!!! :cry: :cry: :cry:


Да потому, что для любого интервала $(a,b)\subseteq\mathbb R$ множество $(a,b)\setminus F_n=(a,b)\cap U_n$ открыто (как пересечение двух открытых множеств) и непусто (так как $U_n$ всюду плотно). Поэтому (по определению открытого множества на числовой прямой) для любой точки $x\in(a,b)\setminus F_n$ (которая обязательно найдётся, так как $(a,b)\setminus F_n$ непусто) существует интервал $(x-\delta,x+\delta)\subseteq(a,b)\setminus F_n$, $\delta>0$, что и означает, что множество $F_n$ нигде не плотно в $\mathbb R$ (по определению нигде не плотного множества).

Вы бы литературу-то почитали бы. Ей богу, пользы больше будет.

 Профиль  
                  
 
 
Сообщение19.11.2005, 07:22 
Хочется уточнить: если множество содержит все рациональные или все иррациональные точки, то оно всюду плотно. Это ведь доказывается по определению всюду плотного множества ветно? Если нет то как? :D :D :D

  
                  
 
 
Сообщение19.11.2005, 17:49 
Заслуженный участник
Аватара пользователя


23/07/05
17973
Москва
Pokemon писал(а):
Хочется уточнить: если множество содержит все рациональные или все иррациональные точки, то оно всюду плотно. Это ведь доказывается по определению всюду плотного множества ветно? Если нет то как? :D :D :D


Исключительно по определению. Поскольку каждый интервал содержит хотя бы одну рациональную и хотя бы одну иррациональную точку, то оба множества всюду плотны на числовой прямой. А если $A\subseteq\mathbb R$ всюду плотно в $\mathbb R$ и $A\subseteq B\subseteq\mathbb R$, то и $B$ всюду плотно в $\mathbb R$ - по определению всюду плотного множества.
Я, вообще-то, нахожусь в некотором затруднении, поскольку не понимаю причины Вашего затруднения в этом вопросе.

 Профиль  
                  
 
 
Сообщение19.11.2005, 20:19 
Someone писал(а):
Pokemon писал(а):
Хочется уточнить: если множество содержит все рациональные или все иррациональные точки, то оно всюду плотно. Это ведь доказывается по определению всюду плотного множества ветно? Если нет то как? :D :D :D


Исключительно по определению. Поскольку каждый интервал содержит хотя бы одну рациональную и хотя бы одну иррациональную точку, то оба множества всюду плотны на числовой прямой. А если $A\subseteq\mathbb R$ всюду плотно в $\mathbb R$ и $A\subseteq B\subseteq\mathbb R$, то и $B$ всюду плотно в $\mathbb R$ - по определению всюду плотного множества.
Я, вообще-то, нахожусь в некотором затруднении, поскольку не понимаю причины Вашего затруднения в этом вопросе.


Мне просто хотелось этот факт уточнить!!! Просто мы ещё не проходили всюду плотное множество! Я изучаю это сам!!! Большое спасибо за помощь!!!

  
                  
 
 
Сообщение19.11.2005, 20:34 
Someone писал(а):
Anonymous писал(а):
А когда мы доказывали что множество иррациональных точек не явл. множ. первой по Бэру категории, сначала писали (Напротив заранее будем предпологать, что множество рац. чисел сод. в этом множ...)! дак вот можно это не писать? Или это объязательно?

А можно этот пункт по-подробней написать? Пожалуйста!!!!!!!!!!
За ранее длагодарен!!! :) :) :)


Нам же нужно найти ИРРАЦИОНАЛЬНОЕ число, не содержащееся в $\bigcup_{n\in\mathbb N}F_n$. Простейший способ обеспечить, чтобы $q_0\in\bigcap_{n\in\mathbb N}[a_n,b_n]$ было иррациональным, состоит в том, чтобы заранее исключить все рациональные числа, присоединив их к $\bigcup_{n\in\mathbb N}F_n$.


Хочется ещё кое что уточнить: мы доказывали что точка q0 является иррациональным числом и она не содержится в множестве $\bigcup_{n\in\mathbb N}F_n$ , то ведь это означает что все иррациональные числа не содержаться в данном множестве? Можете по подробнее этот пункт рассказать? Очень прошу!!!!

  
                  
 
 
Сообщение20.11.2005, 20:42 
Заслуженный участник
Аватара пользователя


23/07/05
17973
Москва
Anonymous писал(а):
Хочется ещё кое что уточнить: мы доказывали что точка q0 является иррациональным числом и она не содержится в множестве $\bigcup_{n\in\mathbb N}F_n$ , то ведь это означает что все иррациональные числа не содержаться в данном множестве? Можете по подробнее этот пункт рассказать?


О, боже! Так мы же и доказываем, что множество иррациональных чисел не содержится в $\bigcup_{n\in\mathbb N}F_n$! Идея доказательства как раз и состоит в том, что мы находим заведомо иррациональную точку $q_0\notin\bigcup_{n\in\mathbb N}F_n$. А заведомо иррациональная она как раз потому, что все рациональные мы включили в $\bigcup_{n\in\mathbb N}F_n$ - я Вам этот момент уже пояснял.

 Профиль  
                  
Показать сообщения за:  Поле сортировки  
Начать новую тему Ответить на тему  [ Сообщений: 76 ]  На страницу Пред.  1, 2, 3, 4, 5, 6  След.

Модераторы: Модераторы Математики, Супермодераторы



Кто сейчас на конференции

Сейчас этот форум просматривают: нет зарегистрированных пользователей


Вы не можете начинать темы
Вы не можете отвечать на сообщения
Вы не можете редактировать свои сообщения
Вы не можете удалять свои сообщения
Вы не можете добавлять вложения

Найти:
Powered by phpBB © 2000, 2002, 2005, 2007 phpBB Group